Question

Question 3: A random variable X has a Bernoulli distribution with parameter θ є (0,1) if X {0,1} and P(X-1)-θ. Suppose that we have nd random variables y, x, following a Bernoulli(0) distribution and observed values y1,... . Jn a) Show that EIX) θ and Var[X] θ(1-0). b) Let θ = ỹ = (yit . .-+ yn)/n. Show that θ is unbiased for θ and compute its variance. c) Let θ-(yit . . . +yn + 1)/(n + 2) (this estimator comes from something called Laplaces rule d) Compute the risk (bias squared plus variance) of θ and Comment on the quality of the two Note: Bias is the difference between the expected value of an estimator and the parameter it is of succession). Show that is based but that it has less variance that θ estimators for θ. estimating

0 0
Add a comment Improve this question Transcribed image text
Answer #1

a)

E[X] = 0 * P(X = 0) + 1 * P(X = 1) = 1 * \theta = \theta

E[X2] = 02 * P(X = 0) + 12 * P(X = 1) = 1 * \theta = \theta

Var[X] = E[X2] - E[X]2  = \theta - \theta^2 = \theta(1 - \theta)

b)

E[\hat{\theta}] = E[\bar{y}] = E[y] = \theta (By central limit theorem, E[\bar{y}] = E[y] )

Thus, \hat{\theta} is unbiased estimator of \theta

Var[\hat{\theta}] = Var[\bar{y}] = Var[y]/n = \theta(1- \theta)/n

c)

E[\tilde{\theta}] = E[(y_1+ ..+y_n+1)/(n+2)] = E[(y_1+ ..+y_n)+1]/(n+2)

= E[n((y_1+ ..+y_n)/n)+1]/(n+2) = E[n\bar{y}+1]/(n+2) = (nE[\bar{y}] + 1)/n+2

E[\tilde{\theta}] = (nE[y] + 1)/n+2 = (n \theta + 1)/(n+2)

As, E[\tilde{\theta}] \ne \theta , \tilde{\theta} is biased estimator of \theta.

Var[\tilde{\theta}] = Var[(y_1+ ..+y_n+1)/(n+2)] = Var[(y_1+ ..+y_n)+1]/(n+2)^2

= Var[n((y_1+ ..+y_n)/n)+1]/(n+2)^2 = Var[n\bar{y}+1]/(n+2)^2

= (n^2Var[\bar{y}])/(n+2)^2 = (n^2 Var[y]/n) / (n+2)^2 = n \theta (1- \theta)/(n+2)^2

Var[\tilde{\theta}] = n^2 \theta (1- \theta)/n(n+2)^2 = (n / n+2)^2 Var[\hat{\theta}]

For n > 1, (n / n+2)2 < 1

thus,

Var[\tilde{\theta}] < Var[\hat{\theta}]

d)

Bias for \hat{\theta} is,

b = E[\hat{\theta}] - \theta = \theta - \theta = 0

Risk = b2 + Var(\hat{\theta}) = (1-0)/11

Bias for \tilde{\theta} is,

b = E[\tilde{\theta}] - \theta =(n \theta + 1)/(n+2) - \theta = ( 1 - 2\theta)/(n+2)

Risk = b2 + Var(\tilde{\theta}) = ( 1 - 2\theta)^2/(n+2)^2 + n \theta (1- \theta)/(n+2)^2 = (( 1 - 2\theta)^2 +n \theta (1- \theta) )/(n+2)^2

For low values of n and large values of \theta, the risk for \hat{\theta} will be larger than that of \tilde{\theta}, but for high values of n, the risk for \hat{\theta} will be lesser than that of \tilde{\theta}.

Add a comment
Know the answer?
Add Answer to:
Question 3: A random variable X has a Bernoulli distribution with parameter θ є (0,1) if...
Your Answer:

Post as a guest

Your Name:

What's your source?

Earn Coins

Coins can be redeemed for fabulous gifts.

Not the answer you're looking for? Ask your own homework help question. Our experts will answer your question WITHIN MINUTES for Free.
Similar Homework Help Questions
ADVERTISEMENT
Free Homework Help App
Download From Google Play
Scan Your Homework
to Get Instant Free Answers
Need Online Homework Help?
Ask a Question
Get Answers For Free
Most questions answered within 3 hours.
ADVERTISEMENT
ADVERTISEMENT
ADVERTISEMENT